Which point is a solution to the inequality shown in this graph?

Which Point Is A Solution To The Inequality Shown In This Graph?

Answers

Answer 1
The answer would be (3,-1), Since the line is solid and not dotted, any point on the line or in the shaded region would be a valid solution.

Related Questions

The first term of an arithmetic sequence is -3 and the fifteenth term is 53. What is the common difference of the
sequence?

Answers

Answer:

53=-3+14d

56=14d

d=4 that is the common difference

PLEASE HELP WILL GIVE 100 POINTS AND BRAINLEST ANSWER BOTH QUESTIONS TWO ANSWERS FOR EACH!!

PLEASE HELP WILL GIVE 100 POINTS AND BRAINLEST ANSWER BOTH QUESTIONS TWO ANSWERS FOR EACH!!

Answers

Circumcenter is equidistant from the vertices, perpendicular bisectors will cross. Incentre is equidistant from the sides. Where the angle bisector cross incenter

What is Circle?

A circle is a shape consisting of all points in a plane that are at a given distance from a given point, the centre.

We have to choose the properties of circumcenter.

Circumcenter is equidistant from the vertices.

It is equidistant from the three vertices of the triangle, and thus you can construct a circle centered at the circumcenter and circumscribing the triangle.

and perpendicular bisectors will cross.

We have to choose the properties of incenter.

Incentre is equidistant from the sides.

Where the angle bisector cross incenter

To learn more on Circles click:

https://brainly.com/question/11833983

#SPJ1

in 2019 a large pizza cost $15.50. Today, a large pizza costs $20. What is the percent increase? Round to the nearest whole percent.

Answers

The percent increase went up 78 percent

determine whether linear or exponential ​

determine whether linear or exponential

Answers

Answers:

The distance driven and the total cost when a taxi driver charges $3 for the first mile and $2 for each additional mile. (LINEAR)The number of bacteria and time when a culture of 5000 bacteria is reduced by 50% every two hours. (EXPONETIAL)The volume of a landfill and time given that the volume triples every five years. (EXPONETIAL)The altitude of a hot air balloon and time when the hot air balloon takes off at 7500 feet above sea level and rises 180 feet every minute. (LINEAR)The cost of the rental space is $780 and the price per person for food is $9.75. (LINEAR)If the half-life of a substance is 2 days, what is the amount of the substance left at the end of 4 days? (EXPONETIAL)

In 1945, a person made $3.20 for working an 8-hour day. model a division problem to determine how much a person made per hour in 1945. what is the hourly wage?

Answers

Answer:

$0.40 per hour

(This is 40cents per hour)

Step-by-step explanation:

The "hourly wage" is a rate, a ratio, a comparing of two numbers. We are comparing dollars and hours. So we have information given in the problem about the $ and hours. Compare these two numbers and then simplify. Because we need the hours, on the bottom, to be 1 hour. "Per hour" means per ONE hour.

dollars/hours

= $3.20/ 8hours

Divide.

= $0.40/1 hour

This is the hourly wage, $0.40 (or 40cents) per hour.

3х + бу
бу =
18 and Зу = - 5х + 9
do these have one solution,ims,or no solution

Answers

Answer:

The solution of the system of equations will be:

\(x=0,\:y=3\)

And the system of equations has ONLY ONE solution.

Step-by-step explanation:

Given the system of the equations

\(\begin{bmatrix}3x+6y=18\\ 3y=-5x+9\end{bmatrix}\)

Arrange equation variables for elimination

\(\begin{bmatrix}3x+6y=18\\ 5x+3y=9\end{bmatrix}\)

\(\mathrm{Multiply\:}3x+6y=18\mathrm{\:by\:}5\:\mathrm{:}\:\quad \:15x+30y=90\)

\(\mathrm{Multiply\:}5x+3y=9\mathrm{\:by\:}3\:\mathrm{:}\:\quad \:15x+9y=27\)

\(\begin{bmatrix}15x+30y=90\\ 15x+9y=27\end{bmatrix}\)

\(15x+9y=27\)

\(-\)

\(\underline{15x+30y=90}\)

\(-21y=-63\)

so the system of the equations becomes

\(\begin{bmatrix}15x+30y=90\\ -21y=-63\end{bmatrix}\)

solve -21y = -63

\(-21y=-63\)

\(\mathrm{Divide\:both\:sides\:by\:}-21\)

\(\frac{-21y}{-21}=\frac{-63}{-21}\)

\(y=3\)

\(\mathrm{For\:}15x+30y=90\mathrm{\:plug\:in\:}y=3\)

\(15x+30\cdot \:3=90\)

\(15x+90=90\)

subtract 90 from both sides

\(15x+90-90=90-90\)

\(15x=0\)

Divide both sides by 15

\(\frac{15x}{15}=\frac{0}{15}\)

\(x = 0\)

as

\(x = 0\), \(y=3\)

so, the system of equations contains only one solution.

Therefore, the solution of the system of equations will be:

\(x=0,\:y=3\)

And the system of equations has ONLY ONE solution.

a survey indicated that the vast majority of college students own their own personal computers what information would you want to know before you accepted the results of this survye

Answers

The required information is (B), (C), (D), and (I).

What is a survey?A survey in human subjects research is a set of questions designed to elicit specific information from a specific group of people. Surveys can be conducted over the phone, by mail, on the internet, or even on street corners or in shopping malls. In fields such as social research and demography, surveys are used to gather or gain knowledge.Survey research is frequently used to assess people's thoughts, opinions, and feelings.

So,

A. Who paid for the survey? It is not required.

B. Who made up the population from which the sample was drawn? Needed.

We need to know if the sample is representative of the "college students" population, on which we are basing our conclusions.

C. What sampling strategy was used? Needed.

We want to know if the sample was chosen at random.

D. How large was the sample size? Needed.

This information is required to determine the accuracy of the results.

E. What are the colleges represented in the sample?

It's not necessary if we already know "B."

F. How many survey questions were there? It is not required.

G. How many people participated in the survey? It is not required.

H. What were the questions? It is not required.

I. Is the language in the questions clear, accurate, unbiased, and valid? Needed.

We want to know if the answer was unbiased and clear so that we can determine if the conclusions are also unbiased.

J. What day of the week was the survey held? It is not required.

Therefore, the required information is B, C, D, and I.

Know more about a survey here:

https://brainly.com/question/14610641

#SPJ4

The correct question is given below:

A survey indicates that the vast majority of college students own their own personal computers. What information would you want to know before you accepted the results of this​ survey? Select all that apply. A. Who funded the​ survey? B. What was the population from which the sample was​ selected? C. What sampling design was​ used? D. What was the sample​ size? E. What are the names of the colleges included in the​ sample? F. How many questions were asked in the​ survey? G. How many people conducted the​ survey? H. What questions were​ asked? I. Were the questions​ clear, accurate,​ unbiased, and​ valid? J. On what day of the week was the survey​ conducted? g

According to Weber's law, two items must differ in weight by ______ percent of weight in order to detect a difference.
A. 5%
B. 10%
C. 20%
D. 50%

Answers

According to Weber's law, two items must differ in weight by "5 percent"of weight in order to detect a difference.

We are given that Weber's law

For weight discrimination, Weber’s law states that the JND is proportional to the weight of the stimulus.

The JND is defined as the smallest difference between two weights that can be detected by an observer.

The proportionality constant is known as Weber’s fraction and varies depending on the type of stimulus and the sensory modality involved.

For weight discrimination, Weber’s fraction is typically around 2-3%.

This means that two items must differ in weight by approximately 2-3% of their weight in order to detect a difference.

The difference of weight in percent is 5%, the correct option is A.

Learn more about percent here:

https://brainly.com/question/11549320

#SPJ4

What is a Jump Discontinuity?


brainliest is on the line and up for grabs

Answers

Answer:

Jump discontinuity is when the two-sided limit doesn't exist because the one-sided limits aren't equal. Asymptotic/infinite discontinuity is when the two-sided limit doesn't exist because it's unbounded.

Image result for What is a Jump Discontinuity?Jump Discontinuity is a classification of discontinuities in which the function jumps, or steps, from one point to another along the curve of the function, often splitting the curve into two separate sections. While continuous functions are often used within mathematics, not al functions are continuous

Step-by-step explanation:

hope this helps

can i have brainliest please

Charlie and Susan are planning a party for 10 people.



Charlie finds a location that charges an initial fee of $20 plus $25 per person.



Susan finds a location whose rental fee is represented by the equation y = 15x + 100, where x is the number of people in attendance and y is the total cost.



Select all of the statements that are true.


Charlie and Susan are planning a party for 10 people.



Charlie finds a location that charges an initial fee of $20 plus $25 per person.



Susan finds a location whose rental fee is represented by the equation y = 15x + 100, where x is the number of people in attendance and y is the total cost.



Select all of the statements that are true.


Charlie's location is a cheaper location.


Susan's Location is cheaper for 10 people.


The charge for each additional person is greater for Susan's location.


The charge for each additional person is greater for Charlie's location.


If the number of people at the party changes to 12, the total cost at each location is the same.

Answers

With the given cost for Charlies location and Susan's linear equation for cost options A, B and D applies.

What is a linear equation, exactly?

When the largest power of a variable is 1, the equation is termed linear.  The standard form of a linear equation with one variable is Ax + B = 0. There are three variables in this equation: x, A, and B. The letter A represents a coefficient. A two-variable linear equation is frequently written as Ax + By = C. The variables x and y, as well as the coefficients A and B, have been added to the constant C.

Now,

As given Cost of Charlie's location=$20+25a where a=no. pf people per person

and Cost of Susan's location is y=15x+100 where x=no. of people

then Cost of 1 person for Charlie and Susan's location are $45 and $115 respectively and

For 10 person  Cost of Charlies location=20+25*10=$270

and Cost of Susan's location for 10 person=15*10+100=$250

Additional charge per person

for charlie=$25

for Susan=$15

Hence,

          Options A, B and D apply.

To know about linear equations visit the link

https://brainly.com/question/29739212

#SPJ1

With the given cost for Charlies location and Susan's linear equation for cost options A, B and D applies.

What is a linear equation, exactly?

When the largest power of a variable is 1, the equation is termed linear.  The standard form of a linear equation with one variable is Ax + B = 0. There are three variables in this equation: x, A, and B. The letter A represents a coefficient. A two-variable linear equation is frequently written as Ax + By = C. The variables x and y, as well as the coefficients A and B, have been added to the constant C.

Now,

As given Cost of Charlie's location=$20+25a where a=no. pf people per person

and Cost of Susan's location is y=15x+100 where x=no. of people

then Cost of 1 person for Charlie and Susan's location are $45 and $115 respectively and

For 10 person  Cost of Charlies location=20+25*10=$270

and Cost of Susan's location for 10 person=15*10+100=$250

Additional charge per person

for charlie=$25

for Susan=$15

Hence,

          Options A, B and D apply.

To know about linear equations visit the link

https://brainly.com/question/29739212

#SPJ1

Besides -10.75+-10.75 what plus what = -21.5?

Answers

Answer:

-32.25 + 10.75 = -21.5

Step-by-step explanation:

-10.75 + -10.75  = -21.5

-32.25 + 10.75 = -21.5

Answer:

4.3+4.3+4.3+4.3+4.3 equals 21.5

According to the empirical rule, the bell or mound shaped distribution will have approximately 68% of the data within what number of standard deviations of the mean

Answers

The correct option is option a) One standard deviation.

According to the empirical rule, the bell or mound shaped distribution will have approximately 68% of the data within one standard deviations of the mean.

According to the empirical rule, the bell or mound-shaped distribution will have approximately 68% of the data within one standard deviation of the mean. This means that if the data is normally distributed, then about 68% of the data points will fall within one standard deviation above or below the mean.

Similarly, the empirical rule states that approximately 95% of the data will fall within two standard deviations of the mean, and about 99.7% of the data will fall within three standard deviations of the mean.

This means that if the data is normally distributed, then 95% of the data points will fall within two standard deviations above or below the mean, and 99.7% of the data points will fall within three standard deviations above or below the mean.

It is important to note that the empirical rule is based on the assumption that the data is normally distributed. If the data does not follow a normal distribution, then the empirical rule may not apply.

Therefore, the answer to the question is (a) One standard deviation, (b) Two standard deviations, and (c) Three standard deviations. Option (d) Four standard deviations and (e) Four standard deviations are not correct, and option (f) None of the above is partially correct as it excludes options (a), (b), and (c), but option (g) All of the above is not correct as options (d) and (e) are incorrect.

To learn more about standard deviations, refer here:

https://brainly.com/question/23907081#

#SPJ11

For the expression startfraction 9 x squared minus 8 over (3 x squared minus 4) squared endfraction, which sum represents the correct form of the partial fraction decomposition?

Answers

To find the correct form of the partial fraction decomposition for the expression:   \(\(\frac{9x^2 - 8}{(3x^2 - 4)^2}\)\)

We need to factor the denominator and decompose it into partial fractions. Let's factor the denominator first:

\(\(3x^2 - 4 = (3x - 2)(x + 2)\)\)

Now, we can decompose the expression into partial fractions using the following general form:

\(\(\frac{A}{3x - 2} + \frac{B}{x + 2} + \frac{C}{(3x - 2)^2} + \frac{D}{(x + 2)^2}\)\)

To determine the values of A, B, C, and D, we need to find the common denominator and equate the numerators:

\(\(9x^2 - 8 = A(x + 2)(3x - 2) + B(3x - 2)(x + 2) + C(x + 2) + D(3x - 2)\)\)

Now, we can solve for A, B, C, and D by comparing the coefficients of like terms on both sides of the equation.

However, by following the steps outlined above, you can determine the specific values and complete the partial fraction decomposition.

Learn more about partial fraction here:

brainly.com/question/30401693

#SPJ11

how do your graph questions like y= 1/2+7

Answers

Answer:

so basically Write a factor

Step-by-step explanation:

1 - write a factor and then multiply then divide that's all hopefully this is helpful:)

.

A lake has a surface area of 15.4 square miles. What is its surface area in square meters?

Answers

The answer should be 3.9886

Use the image to answer the question. A coordinate plane with four quadrants shows the x- and y-axes ranging from negative 5 to 5 in increments of 1. A solid line and a dotted line intersect each other. The equation of the solid line is x minus 5 y equals 3. The equation of the dotted line is 3 x minus 2 y equals negative 4. The intersection of both lines is shown at negative 2 on the x-axis and negative 1 on the y-axis in quadrant 3.

Review the graphs of a system of two linear equations in two variables: x−5y=7 and 3x−2y=−4. Find the solution to both equations.(1 point)
The intersection point is ()

Answers

The equations given are x - 5y = 3 and 3x - 2y = -4. To find the solution to this system of equations, we need to find the values of x and y that satisfy both equations simultaneously.

Find the solution to both equations?

One way to solve this system of equations is by substitution. We can solve one equation for x or y, and then substitute that expression into the other equation to eliminate one variable. Let's solve the first equation for x:

x - 5y = 3

x = 5y + 3

Now we can substitute this expression for x into the second equation:

3x - 2y = -4

3(5y + 3) - 2y = -4

15y + 9 - 2y = -4

13y = -13

y = -1

We can now substitute this value for y back into either equation to find the value of x:

x - 5y = 3

x - 5(-1) = 3

x + 5 = 3

x = -2

Therefore, the solution to the system of equations x - 5y = 3 and 3x - 2y = -4 is (-2, -1). This is the point where the solid line and dotted line intersect, as shown in the image.

to know more about equations

brainly.com/question/29657983

#SPJ1

Identify the domain and range of each function.

y = 3 • 5x

Answers

Answer:

I THINK the domain is

Negative infinity to infinity

and I THINK the range is..

0 to infinity

Step-by-step explanation:

im not sure im sorry if im wrong : ) hope this helps

Answer:

Step-by-step explanation:

negative infinity to infinity

0 to infinity

pt2

negative infinity to infinity

3 to infinity

Tulong bukas na deadlinee plss​

Answers

Answer:

I don't know it like off

Find x:

(Round the answer to the nearest tenth if there is a decimal)

Find x:(Round the answer to the nearest tenth if there is a decimal)

Answers

Answer:Angle x is congruent with the interior angle opposite side 8 (alternate interior angles)

Use tangent:

tan x = 8/15

x = arctan (8/15)

x = 28.1° (rounded)

Step-by-step explanation:

Rewrite as a simplified fraction 3.16

Answers

Answer: 3 4/25

Step-by-step explanation:

A rotationR takes A(1,-3)toA'(3,5) and B(0,0) to B'(4,-6). Find the centre of rotation

Answers

These ratios are not equal, we can conclude that there is no fixed center of rotation for the given transformations. The points A and B do not undergo a simple rotation around a single center.

To find the center of rotation, we need to determine the point around which the rotation occurs.

Let's denote the center of rotation as (h, k).

The rotation takes point A(1, -3) to A'(3, 5) and point B(0, 0) to B'(4, -6).

For point A, the translation in the x-direction is: x' - x = 3 - 1 = 2

For point A, the translation in the y-direction is: y' - y = 5 - (-3) = 8

For point B, the translation in the x-direction is: x' - x = 4 - 0 = 4

For point B, the translation in the y-direction is: y' - y = -6 - 0 = -6

The translations in both the x-direction and y-direction should be proportional for the rotation to occur around a fixed center. This gives us the following ratios:

For the x-direction: (2 / 4) = (8 / -6)

For the y-direction: (2 / 4) = (8 / -6)

Simplifying these ratios, we have:

1/2 = -4/3

Since these ratios are not equal, we can conclude that there is no fixed center of rotation for the given transformations. The points A and B do not undergo a simple rotation around a single center.

Learn more about rotation here:

https://brainly.com/question/1571997

#SPJ11

Sketch a graph of a function, f, that has at least all of the following properties. Draw in any vertical asymptote(s), where appropriate. - lim x→−6 f(x)=3 - lim x→−3f(x)=[infinity] - f(0)=2 - f(x)=0, when x=3 - f ′(3)=0 lim x→7 −f(x)=4 - lim x→7 +f(x)=−5 - lim x→[infinity] f(x)=[infinity]

Answers

The graph of function f has a vertical asymptote at x = -6, passes through the point (0, 2), and has a zero at x = 3. As x approaches -3, the function approaches infinity. The derivative of the function at x = 3 is zero. As x approaches 7 from the left, the function approaches 4, and as x approaches 7 from the right, the function approaches -5. As x approaches infinity, the function approaches infinity.  

The vertical asymptote at x = -6 indicates that the function approaches infinity or negative infinity as x approaches -6. The point (0, 2) means that the function passes through the y-axis at y = 2. The zero at x = 3 means that the function crosses the x-axis at x = 3. The limit as x approaches -3 being infinity indicates that the function grows without bound as x approaches -3. The derivative of the function at x = 3 being zero suggests that the function has a horizontal tangent at that point. The limits as x approaches 7 from the left and right indicate that the function approaches different values from each side, resulting in a jump discontinuity. Finally, the limit as x approaches infinity being infinity implies that the function grows without bound as x becomes larger.

Learn more about derivative here:

https://brainly.com/question/29020856

#SPJ11

actoring Quadratic Expressions. Factor each completely. 1) x. 2 − 7x − 18. 2) p. 2 − 5p − 14. 3) m. 2 − 9m + 8.

Answers

Completely factored expressions are:

(x - 9)(x + 2)(p - 7)(p + 2)(m - 1)(m - 8)

How to evaluate each part of the question?

1. x² - 7x - 18 can be factored as:

(x - 9)(x + 2)

Expand the expression using FOIL:

(x - 9)(x + 2) = x² + 2x - 9x - 18 = x² - 7x - 18

2. p² - 5p - 14 can be factored as:

(p - 7)(p + 2)

Expand the expression using FOIL:

(p - 7)(p + 2) = p² + 2p - 7p - 14 = p² - 5p - 14

3. m² - 9m + 8 can be factored as:

(m - 1)(m - 8)

Expand the expression using FOIL:

(m - 1)(m - 8) = m² - 8m - m + 8 = m² - 9m + 8

Therefore, the completely factored expressions are:

(x - 9)(x + 2)(p - 7)(p + 2)(m - 1)(m - 8)

Learn more about factored expressions.

brainly.com/question/19386208

#SPJ11

calculate the amount that will be in the bank after 4years if R3500 was invested at 9% p.a. simple interest​

Answers

Answer:

4760

Step-by-step explanation:

Simple interest= principal× time ×rate/100

=3500×4×9/100

= 1260

Amount that will be in the bank after 4years= 3500+1260

=4760

william is 3 feet 1 inch tall and would like to ride a roller coaster. riders must be at least 42 inches tall to ride the coaster. write an addition inequality to determine how much taller william must be to ride the coaster. let x be the variable representing how much taller william must be.

Answers

The addition inequality 37 + x ≥ 42 can be used to determine how much taller William must be to ride the coaster, where x represents the additional height he needs to meet the height requirement.

To write an addition inequality to determine how much taller William must be to ride the coaster, we first need to convert his height to inches. Since he is 3 feet 1 inch tall, his height is (3 x 12) + 1 = 37 inches.

Let x be the amount of additional height William needs to ride the roller coaster. The inequality can be written as:

37 + x ≥ 42

This inequality ensures that William's total height after adding x must be greater than or equal to the required height of 42 inches to ride the roller coaster.

To learn more about inequality : brainly.com/question/20383699

#SPJ11

Which expenses for the month of July had a negative variance we’re more than the amount budgeted

Which expenses for the month of July had a negative variance were more than the amount budgeted

Answers

For the fixed expenses, Penders spends $150 less than the budgeted amount.

What is a Budget?

A budget is a written plan that you make to decide how much money you will spend each month. A budget can help you ensure that you have enough money each month. If you don't have a budget, you may run out of money before your next payday. Your budget reveals how much money you make.

The three major budgets are as follows:

Budget that is balanced. A balanced budget is one in which the expected government expenses equal the estimated government receipts during a given fiscal year.

Budget Surplus. The surplus budget is the second of the three types of budgets.

Budget Deficit.

Read more about budgets here:

https://brainly.com/question/6663636

#SPJ1

Which expenses for the month of July had a negative variance were more than the amount budgeted
Which expenses for the month of July had a negative variance were more than the amount budgeted

9th grade math HHhH HHhH

9th grade math HHhH HHhH

Answers

Answer:y=-x+3

Step-by-step explanation:

What is the volume of this prism?

What is the volume of this prism?

Answers

Answer:

47.7

Step-by-step explanation:

length x width x height

6 x 3 x 2.65

When Jack runs the 400 meter dash, his finishing times are normally distributed with a mean of 65 seconds and a standard deviation of 1 second. If Jack were to run 20 practice trials of the 400 meter dash, how many of those trials would be faster than 66 seconds, to the nearest whole number?

Answers

The trials would be faster than 66 seconds in 3 trials.

Probability: The ratio of the number of outcomes in an exhaustive set of equally likely outcomes that produce a given event to the total number of favorable outcomes.

z-score = (x – μ) / σ------(1)

x: individual data value

μ: population mean

σ: population standard deviation

Given the information in the question is

standard deviation =1

mean(u)=65 x=66 n=20

substitute the values in eq(1)

Z=x-u/sd

Z =66=65/1

Zat 1=0.84134

The probability is faster than 10 seconds = 1-Z

                                                                     =(1-0.84134)

                                                                     P=0.15866

A number of trials=n(0.15866)

                             = 20(0.15866)

                           Nt =3.17

The nearest whole number is 3.

To know more about the Probability of Z-score:

https://brainly.com/question/28096232

#SPJ4

Suppose there is a 1.8 Farenheit drop in temperature for every thousand feet that an airplane climbs into the sky. If the temperature on the ground is 62.4 farenheit ​, what will be the temperature when the plane reaches an altitude of 8,000 ft. ​?

Answers

Answer:

48° F

Step-by-step explanation:

Let the altitude on the ground be 0 ft

Temperature at 8000 ft is:

62.4 - 1.8°*(8000 - 0)/1000 = 62.4 - 1.8*8 =62.4 - 14.4 =48° F

Answer:

anyone seen joel

Step-by-step explanation:

Other Questions
_________ is what the conversation is abouta.businessb.impersonal communicationc.closing arguments d.subject Which individual interferes with or disrupts systems to protest the operations, policies, or actions of an organization or government agency 32-year-old woman presents to your office for a physical exam including a papanicolaou test (pap smear). lab results reveal negative cytology and positive human papillomavirus (hpv). which of the following is the most appropriate next step in management? if a home garden were planted with all hybird round peas, what phenotypic proportions would be observed in the progency please help me its due nowww What is the domain of this function? A. {-3,0,3}B. {6,7}C. {7} D. {-3,0,3,6,7} which statement best describes how the author develop the central idea of the passage?IREADY the nurse is teaching a parent about promoting safety in adolescents. which statements made by the parent indicate the need for further teaching? select all that apply. one, some, or all responses may be correct. Suppose y varies inversely with x, and y = 36 when x=112What inverse variation equation relates x and y ? In the right triangle shown in the diagram below,what is the value of x to the nearest whole number? A podcast fan club charges a one-time enrollment fee and a monthly subscription fee for each month you are signed up. What is the cost per month to be in the fan club? _________.What does the first point (0, 34) represent in real-life terms? _________.Using m to represent months and C(m) to represent the total cost, what equation models the function in the table? _______.Is this function linear? _________. i have no clue what this answer is When we view the overall emphasis of the Bible, the primary view indicated by the scriptures is _______________. a. that the Bible is redemptive story, told by God as author and main character b. that the Bible provides a roster of role models to emulate c. that the Bible offers a list of rules to follow d. that the Bible serves as a practical guide for a successful life Is the hearsay exception impending death able to be used in an attempted murder case, if so why? Phobos and Deimos are ... Group of answer choices just 0.3% of the size of Mars the same size as Mars half as big as Mars twice as big as Mars dwarf planets Guys help me I need to write a function rule but the equation format is mx+b 2y 7x + 4x2 + 3y + 7xA 4x2 14x + 5yB 4x2 14xC 4x2 + 14x + 5yD 4x2 + 5y Angle B and angle C are supplementary. If the measure of angle B is 14 times the measure of angle C, what is the difference between the angles' measures?Enter your answer as the correct number of degrees, like this: 42explain why Tamara is a chef at a restaurant. She buys rice in bags that weigh 20 pounds.Each night, Tamara uses 0,4 pounds of rice. She needs toorder another bag when there are 1.6 pounds of rice leftin her bag.Question: On Day 1 there are 20 poundsof rice in her bag. On what day willTamara need to order a new bag of rice? If you discover a new type of muscle fiber that has a high concentration of myoglobin, many mitochondria and an abundant supply of blood, you would expect the fiber to be a: